Calculus 10th Edition

Published by Brooks Cole
ISBN 10: 1-28505-709-0
ISBN 13: 978-1-28505-709-5

Chapter 9 - Infinite Series - 9.2 Exercises - Page 601: 3

Answer

$S_{1}=3;$ $S_{2}=S_{1}-\dfrac {9}{2}=-\dfrac {3}{2};$ $S_{3}=S_{2}+\dfrac {27}{4}=\dfrac {21}{4};$ $S_{4}=S_{3}-\dfrac {81}{8}=-\dfrac {39}{8};$ $S_{5}=S_{4}+\dfrac {243}{16}=\dfrac {165}{16}$

Work Step by Step

$S_{n}=3-\dfrac {9}{2}+\dfrac {27}{4}-\dfrac {81}{8}+\dfrac {243}{16}\ldots $ $S_{1}=3;$ $S_{2}=S_{1}-\dfrac {9}{2}=-\dfrac {3}{2};$ $S_{3}=S_{2}+\dfrac {27}{4}=\dfrac {21}{4};$ $S_{4}=S_{3}-\dfrac {81}{8}=-\dfrac {39}{8};$ $S_{5}=S_{4}+\dfrac {243}{16}=\dfrac {165}{16}$
Update this answer!

You can help us out by revising, improving and updating this answer.

Update this answer

After you claim an answer you’ll have 24 hours to send in a draft. An editor will review the submission and either publish your submission or provide feedback.